LSAT and Law School Admissions Forum

Get expert LSAT preparation and law school admissions advice from PowerScore Test Preparation.

 Administrator
PowerScore Staff
  • PowerScore Staff
  • Posts: 8917
  • Joined: Feb 02, 2011
|
#38160
Complete Question Explanation
(The complete setup for this game can be found here: lsat/viewtopic.php?t=14979)

The correct answer choice is (D)

For the first time ever, test-makers decided to include two Rule Substitution questions in the Logic Games section, of which this is the second one (the first one was Question #5 in Game 1). Considering their growing popularity, tackling such questions is no longer merely optional: it is imperative to have a workable strategy in place.

The question stem requires you to replace the fourth rule of the game, which created the following relationship:
PT71_D13 LG Explanations_Game #3_#16_diagram 1.png
Thankfully, the rule only concerns the order in which the books are being discussed (regardless of whether they are summarized or not). Without this rule, we are left with the following sequencing relationship:
  • ..... ..... ..... ..... ..... ..... N :longline: T :longline: O
Our job is to substitute the fourth rule with a logically equivalent condition, which would have the exact same effect on the order in which books are being discussed. While the wording of the correct answer can be difficult-to-impossible to predict, the four incorrect answer choices will either present rules whose effects are extraneous to the original rule set (also known as Additional Effects conditions), or else they will only partially constrain the variables in the rule being substituted (i.e. Partial Match conditions).

Since Additional Effects are easier to spot, look for answer choices that contain such conditions first. You should immediately notice, for instance, that answer choices (A), (B), and (E) add conditions extraneous to the original rule set. Answer choice (A) stipulates that T must be discussed third, which is not necessarily true (T can also be discussed second). Likewise, answer choice (B) requires T to be discussed earlier than F, which is not an inference we can make using our original diagram. Finally, answer choice (E) requires F to be discussed third. While F must be among the first three books discussed, it is not necessarily third. Thus, we can safely eliminate these Additional Effects answer choices.

We are left with answer choices (C) and (D). According to answer choice (C), K and R are among the last three books discussed, and F is among the first three books discussed. Both conditions are true given our original diagram; however, they do not have the exact same effect on the order in which the books are discussed. Since K and R are only required to be among the last three (not two) books discussed, O is no longer constrained to occupy the fourth position. Indeed, O can be fourth, fifth, or sixth—in violation of our original diagram! Therefore, answer choice (C) contains a Partial Match, and must be eliminated.

Answer choice (D): This is the correct answer choice. Both conditions in this answer choice—K and R are discussed consecutively, and O is discussed fourth—are consistent with the original diagram. Answer choice (D) does not therefore contain an Additional Effects rule. Answer choice (D) is not a Partial Match either: the conditions force K and R to be the last two books discussed, because if they were discussed earlier than O, we would not be able to accommodate the N :longline: T :longline: O sequence. Thus, the effect is identical, and this is the correct answer choice.
You do not have the required permissions to view the files attached to this post.
 sdusso
  • Posts: 6
  • Joined: Jul 08, 2014
|
#16946
I am still really struggling with rule substitution questions. I've gone over the material in the logic games bible and have done many of them on practice tests but for some reason I'm never able to make the inference I need to when I am taking a practice test. Any ideas?
 Lucas Moreau
PowerScore Staff
  • PowerScore Staff
  • Posts: 216
  • Joined: Dec 13, 2012
|
#16947
Hello, sdusso,

I'll tell you what I told eober, who also made this topic: Rule substitution questions are the worst! :x They require more work than almost any other type of question. I recommend creating a new sub-diagram with the different rules in place, if you have time, as that will lead you towards the right answer.

But if you can, try to ask yourself - in what way did the rule that was taken out affect the game? Did it cause a certain block to come into existence? Did it limit where a certain element could be placed? What effect does this new rule have, is it similar to or different from the old rule?

Also, consider what type of question it is. If it's Must Be True/False, look to elements that are now due to the new rules much more constrained in their placement. If it's Could Be True/False, look for elements that suddenly have more places they could be in.

Hope that helps,
Lucas Moreau
 sdusso
  • Posts: 6
  • Joined: Jul 08, 2014
|
#16981
Thank you that does help! If you don't mind me asking one more question. I have been practicing and getting better at them but I am stumped by the 3rd game on the December 2013 practice test. For question 16 I chose answer A and the correct answer is D. They both include different variables but appear to have the same effect. Can you explain why D is a better choice?
 David Boyle
PowerScore Staff
  • PowerScore Staff
  • Posts: 836
  • Joined: Jun 07, 2013
|
#16996
sdusso wrote:Thank you that does help! If you don't mind me asking one more question. I have been practicing and getting better at them but I am stumped by the 3rd game on the December 2013 practice test. For question 16 I chose answer A and the correct answer is D. They both include different variables but appear to have the same effect. Can you explain why D is a better choice?
Hello sdusso,

They actually may not have the same effect. Answer A traps t in third, though normally, it could be either second or third. Answer D, however, works, because with o in 4th, and k and r having to be consecutive (a block), the kr block cannot go ahead of o, since n and t also have to go ahead of o, and there isn't room.

Hope this helps,
David
 medialaw111516
  • Posts: 80
  • Joined: Dec 11, 2018
|
#71905
I hate these questions because I'm so confused by them, and I feel like I'm just wasting time making mini diagrams for each new option, but here I got closer than I usually do, narrowed it down to D and E, but picked E instead of D. I felt likt this was saying the same thing, but is D correct because it explicitly states that O is 4th which tells you in which end of the diagram K and R have to be?
User avatar
 KelseyWoods
PowerScore Staff
  • PowerScore Staff
  • Posts: 1079
  • Joined: Jun 26, 2013
|
#71950
Hi medialaw111516!

Remember that for this question we are being asked to replace a rule with a new rule that would still result in the exact same setup. Answer choice (E) is incorrect because it states that F must be 3rd, which is not true in our Global diagram (F must just be somewhere in the first 3 spaces). O must be 4th in our Global diagram, however, so (D) is correct.

Check out the Global diagram here to make sure you caught all of the inferences: https://forum.powerscore.com/lsat/viewtopic.php?t=14979

Hope this helps!

Best,
Kelsey
 angelsfan0055
  • Posts: 39
  • Joined: Feb 26, 2021
|
#91596
Is the reason why D is correct because when you do a hypothetical, O gets put 4th, and then due to the third rule, K and R have to go 5th and sixth because that's the only place they can go consecutively?

I chose C, an answer I didn't love, in part because of the last [three /i] books discussed instead of 2, but I thought that answer choice D was adding an additional effect. Once I did they hypothetical after reading the explanation that makes more sense.

Is this understanding correct?
 Adam Tyson
PowerScore Staff
  • PowerScore Staff
  • Posts: 5153
  • Joined: Apr 14, 2011
|
#91630
It's more than just one hypothetical solution, angelsfan - it's a global inference that should be made during the setup of the game that O is ALWAYS 4th and K and R are ALWAYS the last two in one order or the other. Answer D forces all those same things to happen, including forcing F to be before O somewhere, while answer C would allow for different outcomes, such as O being 5th or 6th.

The goal for answering a Rule Substitution question like this ("have the same effect") is to select an answer that results in the same main diagram as the original rule did, including all the same inferences and the same answers to all of the questions. If the answer allows for any variation from that original setup, it's wrong!

Get the most out of your LSAT Prep Plus subscription.

Analyze and track your performance with our Testing and Analytics Package.